PANCE exam F part 1

Ace your homework & exams now with Quizwiz!

A 4-week-old infant presents in cardiac arrest. Cardiopulmonary resuscitation is initiated, and the monitor shows pulseless electrical activity. What is the correct concentration and dose of intravenous epinephrine in this setting? A0.01 mL/kg of 1:1,000 concentration B0.01 mL/kg of 1:10,000 concentration C0.1 mL/kg of 1:1,000 concentration D0.1 mL/kg of 1:10,000 concentration

0.1 mL/kg of 1:10,000 concentration ---------------------------------------------------------------- vs Using 0.01 mL/kg of 1:1,000 concentration (A) would be the correct dose and concentration in the setting of anaphylaxis. However, it should be given intramuscularly. 0.01 mL/kg of 1:10,000 concentration (B) is the correct concentration but is not the correct dose. 0.1 mL/kg of 1:1,000 concentration (C) would be the correct dose and concentration in this setting if the medication was being administered via an endotracheal tube.

An 18-year-old man presents with lower extremity weakness for 4 days. He states he had a bout of diarrhea 2 weeks ago that resolved, but then he began feeling weak in his legs. Over the last 24 hours, he has begun to feel weak in his arms and has difficulty breathing. Physical examination reveals 3/5 strength in both lower extremities and 4/5 strength in both upper extremities, along with decreased deep tendon reflexes. What management is indicated for the suspected diagnosis? ACeftriaxone and vancomycin BIntravenous immunoglobulin CMRI of the brain DOral steroids A 60-year-old man is admitted to the general medicine floor. He presented to the emergency department 1 week ago with bilateral leg weakness that had worsened to include bilateral arm and hand weakness after recovering from a recent gastrointestinal infection. You are performing your daily rounds. The patient reports feeling well this morning. On exam, he has a T of 97°F, RR of 18/min, oxygen saturation of 99% on room air, HR of 85 bpm, and BP of 117/77 mm Hg. The patient has not had trouble breathing independently. Based on the suspected diagnosis, what is the most likely exam finding? ABilateral papilledema BCogwheel rigidit

1) Intravenous immunoglobulin 2)Hyporeflexia dx:Guillain-Barré Syndrome Risk Factors: recent minor respiratory or GI illness Sx: Symmetric, progressive ascending muscle weakness, can lead to respiratory failure, often associated with dysautonomia PE: lack of deep tendon reflexes, symmetric weakness Lumbar puncture: increased CSF protein but a normal cell count Most commonly caused by Campylobacter jejuni Treatment is supportive, IVIG, or plasmapheresis

Which of the following patients with atrial fibrillation has the greatest risk for complications from anticoagulation therapy? AA 64-year-old woman with hypertension and history of colon cancer BA 66-year-old man with a history of hypertension, diabetes, and ethanol use disorder CA 79-year-old man with a history of heart failure DA 90-year-old otherwise healthy woman

A 66-year-old man with a history of hypertension, diabetes, and ethanol use disorder

Which of the following patients most likely has dementia? AA confused 65-year-old man brought from a house fire who exhibits good long-term recall, no recall of immediate events, normal vital signs, and carboxyhemoglobin level of 7% BA confused 65-year-old man brought from a house fire who exhibits poor long-term recall, no recall of immediate events, normal vital signs, and carboxyhemoglobin level of 25% CA confused 80-year-old man brought from home with fever, back pain, and urinary retention DA confused 80-year-old man brought from home with fever, cough, and hypoxia EA confused 80-year-old man brought from home with increased thirst, increased urination, and blood sugar monitor that reads high

A confused 65-year-old man brought from a house fire who exhibits good long-term recall, no recall of immediate events, normal vital signs, and carboxyhemoglobin level of 7%

Which of the following is the most common cause of small bowel obstruction in a 70-year-old man with a history of an appendectomy 50 years ago? AAdhesions BIncarcerated hernia CIntussusception DNeoplasm

Adhesions Small Bowel Obstruction History of prior abdominal or pelvic surgery Bilious vomiting PE may show high-pitched bowel sounds X-ray will show dilated bowel, air fluid levels, stack of coins or string of pearls sign Diagnosis is made by imaging Treatment is NPO, IVF, NGT, surgery

A 60-year-old man with a history of intravenous drug use presents to the emergency department with complaints of inability to open his mouth and difficulty swallowing. A review of his medical records shows that he is not up-to-date on his immunizations. On physical examination, stimulation of his posterior pharynx causes him to bite down instead of gag. You also note areas of inflammation, erythema and purulent discharge in bilateral antecubital fossae. Which of the following is the most appropriate management? AAdmission to the intensive care unit BConsult with otolaryngology CTracheostomy DWound debridement

Admission to the intensive care unit dx: Tetanus

A 7-year-old boy presents to your office for a well-child check. His mother has concerns regarding his car booster seat. Which of the following is the most accurate statement regarding car safety seats? AAll children should be restrained in the rear seats of vehicles for optimal protection until they have outgrown the highest weight or height allowed by the manufacturer of their belt-positioning booster seat, typically after 12 years of age. BAll children should be transitioned to belt-positioning booster seats as early as possible for optimal protection. CAll children should use a belt-positioning booster seat until the vehicle lap-and-shoulder seat belt fits properly, typically when they have reached 4 feet 9 inches in height and are between 8 and 12 years of age. DAll infants and toddlers should ride in a rear-facing car safety seat until they are 1 year of age or older.

All children should use a belt-positioning booster seat until the vehicle lap-and-shoulder seat belt fits properly, typically when they have reached 4 feet 9 inches in height and are between 8 and 12 years of age.

A 21-year-old previously healthy man presents to your office with a complaint of severe cold-like symptoms. He says that he started to feel sick two weeks ago. He thought he was getting better, and then on day seven, his symptoms significantly worsened, including sinus congestion, facial pressure, postnasal drip and temperature of 102°F. Physical exam reveals tenderness to palpation of frontal and maxillary sinuses. Which of the following is the most appropriate therapy? AAmoxicillin-clavulanate BAzithromycin CCeftriaxone DTrimethoprim-sulfamethoxazole

Amoxicillin-clavulanate Acute Sinusitis Sx: nasal congestion, pain or pressure over sinuses, ear pain or pressure, headache, fever PE: purulent rhinorrhea Most commonly caused by viral URI If viral, tx: supportive care Bacterial sinusitis: purulent nasal secretions and severe symptoms for ≥ 10 daysAmoxicillin-clavulanate Complications: frontal bone osteomyelitis (Pott puffy tumor), orbital cellulitis, sinus venous thrombosis, extension into meninges/brain

A 25-year-old G1P0 woman at 33 weeks gestation presents to her obstetrician with a complaint of wetness and leakage from her vaginal area. On physical exam, fluid can be seen coming from the cervical canal. The cervix is not dilated. Fern test is positive. Which of the following medications should be given at this time to delay delivery? AAmpicillin BBetamethasone CIndomethacin DProgesterone

Ampicillin dx: A significant proportion of preterm births are complicated by prelabor rupture of membranes. Once preterm, prelabor rupture of membranes is confirmed, ampicillin, a penicillin antibiotic, is given to decrease infection in the mother and the infant thereby delaying preterm labor. It is recommended that azithromycin 1 gram orally followed byampicillin 2 grams intravenously every 6 hours for 48 hours, this is then followed by a course of oral amoxicillin for 5 days. Prelabor Rupture of Membranes (PROM) Membrane rupture prior to labor Preterm PROM (PPROM): PROM occurring at < 37 weeks Fluid ferning + blue nitrazine paper = amniotic fluid Admission, OB consultation ----------------------------------------------------------------- vs Betamethasone (B), an antenatal corticosteroid, is administered to promote fetal lung maturity and to reduce the risk of other complications of prematurity such as necrotizing enterocolitis and intraventricular hemorrhage, not to delay delivery. Indomethacin (C), a nonspecific cyclooxygenase inhibitor, is considered a first-line tocolytic for gestational age between 24 and 32 weeks gestation but is not appropriate at 33 weeks gestation due to the risk of premature closure of the ductus arteriosus. Previously weekly intramuscular progesterone (D) injections were administered from 16 to 36-37 weeks gestation in women with a history of prior preterm birth to reduce the risk of recurrence but should be discontinued after membrane rupture. This practice is still seen in some areas but there is little evidence to support its continued use.

An 8-year-old boy presents with a complaint of excessive thirst and urination. His mother comments that he sleeps "all the time". Which of the following test result scenarios is needed to make a diagnosis of type 1 diabetes mellitus? AFingerstick glucose >126 mg/dL, taken between 5am and 7am BSerum hemoglobin-A1c of 6.1%, drawn upon first presentation of polyuria, polydipsia or polyphagia CTwo-hour serum glucose >200 mg/dL, checked after a 75g oral glucose tolerance test DUrine ketones >2 mg/dL, sampled once between the ages of 5 and 10 years

Two-hour serum glucose >200 mg/dL, checked after a 75g oral glucose tolerance test Random plasma glucose ≥ 200 mg/dL Fasting plasma glucose ≥ 126 mg/dL Glycated hemoglobin (A1C) ≥ 6.5% Plasma glucose ≥ 200 mg/dL 2 hours after a 75 g glucose load during an oral glucose tolerance test Most commonly caused by autoimmune destruction of pancreatic cells Treatment is insulin

A 4-year-old boy presents to the emergency department with right knee pain and swelling after he tripped and fell at home. His parent states that he was walking before he tripped over a toy and fell on his right side, and his knee started swelling immediately. His temperature is 98.9°F, blood pressure is 115/78 mm Hg, heart rate is 105 bpm, oxygen saturation is 99% on room air, and respiratory rate is 24/min. A physical exam shows moderate edema over the right knee that is mildly tender to palpation with decreased range of motion. Laboratory findings reveal a hemoglobin of 12.5 g/dL, a platelet count of 250 × 109/L, and a WBC count of 6,000/µL. A knee X-ray shows a large effusion but no evidence of fracture. What is the best next step in the management of this patient's condition? AAspiration of fluid in the knee BCoagulation studies CComplete skeletal survey DComputed tomography of the right knee EReassurance and observation

Coagulation studies dx: Hemophilia

What imaging test is most helpful in the workup of vertebrobasilar insufficiency?

Answer: MRI of the brain.

A 42-year-old woman presents to the office reporting pain in her fingertips that has worsened over the past year. She notices it when the weather is cold or when she is anxious. When it occurs, her fingertips become pale and numb, and after she warms her hands, the same fingers become bright red with painful numbness and tingling throughout that lasts for several minutes before resolving. She also reports having increased epigastric pain with a burning sensation and a bitter taste in her mouth, especially after meals. She has been taking over-the-counter antacids, which help. Which of the following antibodies are most likely to be found on a blood test? AAnti-cyclic citrullinated peptide antibodies BAnti-double stranded DNA antibodies CAnti-Sjögren syndrome A antibodies and anti-Sjögren syndrome B antibodies DAnti-Smith antibodies EAnticentromere antibodies

Anticentromere antibodies dx: CREST

A 26-year-old woman complains of sneezing, nasal congestion and ocular pruritus. She just moved into an apartment that was previously inhabited by pet owners. You prescribe intranasal corticosteroids, but two weeks later, she returns with no improvement. Which of the following is the most appropriate treatment option at this time?😨😨 AAzelastine BDust mite-proof pillowcases CHydroxyzine DIn-room air filtration

Azelastine dx: Allergic rhinitis is an IgE mediated illness. Symptoms include nasal congestion, rhinorrhea, sneezing and nasal and ocular pruritus Avoidance of known allergens is crucial. First-line therapy is intranasal corticosteroids. Patients who are refractory, or have initial moderate to severe symptoms, should be treated with intranasal irrigation, decongestants, intranasal or oral antihistamines, such as azelastine, cromolyn or leukotriene receptor antagonists. Immunotherapy is considered in those who do not respond to pharmacologics. ------------------------------------------------------------ vs Air-filters (D) and mite-proof pillowcases (B) have not been shown to be effective in allergic rhinitis. Hydroxyzine (E), a first-generation oral antihistamine is not prefered over a second-generation agent since first-generation antihistamines have effects on the central nervous system.

Which of the following is the most effective initial treatment for a benign esophageal stricture? ABalloon dilation BBalloon dilation and proton pump inhibitor CEsophageal stent DNissen fundoplication

Balloon dilation and proton pump inhibitor dx: most effective initial treatment for a benign esophageal stricture. An esophageal stricture is the narrowing of the esophagus caused by the buildup of acid and is one of the complications of gastrointestinal reflux disease. Esophageal Stricture Most common cause: GERD Progressive solid food dysphagia Diagnostic studies include upper endoscopy History or symptoms of a proximal esophageal lesion (e.g., Zenker diverticulum, surgery for laryngeal or esophageal cancer) or known complex stricture: initial test should be preendoscopy barium esophagram Surgery, high-dose PPIs

A 46-year-old woman with a past medical history of chronic back pain is being admitted to the surgery floor after sustaining a femur fracture in a motor vehicle collision. The patient is currently rating her pain a 10/10. At home, she is taking 30 mg of extended-release oxycodone twice daily and uses an average of three doses of 10 mg immediate-release oxycodone per day for breakthrough pain. Which of the following would be the most appropriate order for intravenous morphine via patient-controlled analgesia? ABasal rate: 0 mg/hr, demand dose: 2 mg, lockout interval: 10 minutes BBasal rate: 2 mg/hr, demand dose: 1 mg, lockout interval: 10 minutes CBasal rate: 4 mg/hr, demand dose: 2 mg, lockout interval: 10 minutes DBasal rate: 6 mg/hr, demand dose: 3 mg, lockout interval: 10 minutes

Basal rate: 2 mg/hr, demand dose: 1 mg, lockout interval: 10 minutes step 1: determine their daily total oral morphine milligram equivalents. According to the Centers for Disease Control, oral oxycodone can be converted in a 1:1.5 ratio to oral morphine. This patient takes an average of 90 mg of oxycodone per day, which converts to (90 mg x 1.5 morphine milligram equivalents) 135 mg of oral morphine per day. step 2: Oral morphine is then converted to its IV equivalent at a ratio of 3:1, as recommended by the American Pain Society. Therefore, 135 mg of oral morphine per day converts to (135 mg/3) 45 mg IV morphine per day step 3: This daily amount of IV morphine is then divided by 24 hours per day to get an hourly rate of 1.9 mg/hr, which can be rounded to 2 mg/hr. step 4 :A general rule of thumb for the demand dose of a patient-controlled analgesia is half of the hourly basal rate, which would be 1 mg per dose

What is the most common cause of vertigo? ABenign paroxysmal positional vertigo BLabyrinthitis CMeniere disease DVertebrobasilar insufficiency

Benign paroxysmal positional vertigo

A previously healthy 18-year-old woman presents with sore throat and pain with swallowing. Her vital signs are T 102.7°F, HR 124 bpm, BP 123/76 mm Hg, RR 22/min, and oxygen saturation 97%. On examination, she has trismus, pain with neck extension, and difficulty swallowing her saliva. Her oropharyngeal examination is unremarkable. Which of the following is the most appropriate next step in management? ACT scan of the neck with contrast and ENT consultation BIbuprofen, dexamethasone, and a rapid strep test COral antibiotics and ENT follow-up DPeritonsillar needle aspiration

CT scan of the neck with contrast and ENT consultation dx: retropharyngeal abscess Patient will be a toxic-appearing child, 3-5 years old History of trauma or URI Fever, sore throat, dysphagia PE will show trismus, stridor, nuchal rigidity, muffled voice, cri du canard (duck quack) Neck X-ray will show widened retropharyngeal space twice the size of the vertebral body Diagnosis is made by CT Most commonly caused by S. aureus, group A Streptococcus, anaerobes, foreign body Treatment is intravenous antibiotics, I&D What is the normal width of the retropharyngeal space on lateral neck X-ray? Answer: 7 mm (at C2) in both children and adults.

A 76-year-old woman presents to her primary care physician due to increasing chest pain and shortness of breath at rest. She has a history of hypertension and states that she was told 3 months ago that she has a murmur but cannot recall any other information about this diagnosis. Which of the following is the most likely diagnosis in this patient? ABicuspid aortic valve BCalcified aortic valve CMitral stenosis DTricuspid regurgitation

Calcified aortic valve This woman has aortic stenosis. The prevalence of valvular disease is increasing with age Risk factors: advancing age, diabetes, hypertension Sx: dyspnea, chest pain, syncope PE: crescendo-decrescendo systolic murmur that radiates to the carotids, paradoxically split S2, S4 gallop. Murmur decreases with Valsalva Most commonly caused by degenerative calcification Treatment: aortic valve replacement

A 32-year-old man presents to the emergency department with complaints of a headache. It started gradually earlier today and is associated with nausea and dizziness. He has had similar headaches in the past, but this one is not improving with ibuprofen and acetaminophen. He states he had to turn the heat on the night prior because it was so cold. He feels like he is beginning to feel a little better after coming in. Vital signs are within normal limits. Which of the following diagnostic studies is most likely to confirm the diagnosis? AArterial blood gas BCarboxyhemoglobin CComputed tomography of the brain DLumbar puncture

Carboxyhemoglobin Carbon Monoxide (CO) Poisoning Risk factors: smoke inhalation, fuel-burning heaters Sx: headache, nausea, and dizziness PE: altered mental status, cherry-red skin (postmortem finding) Pulse oximetry does not distinguish between carboxyhemoglobin and oxyhemoglobin Labs: ABG - carboxyhemoglobin Treatment is 100% oxygen, hyperbaric oxygen in some high risk patients (ex coma, pregnancy)

A 2-year-old boy is being evaluated for dysphagia and difficulty breathing. His mother tells you his symptoms suddenly began about 3 hours ago. His immunizations are up to date. He appears restless and irritable, and is drooling. His temperature is 40.0°C (104°F). Physical examination shows suprasternal and subcostal retractions. Auscultation of the lungs reveals inspiratory stridor. Blood cultures are pending. Which of the following is the most appropriate treatment for this patient's condition? AAlbuterol BAmpicillin CCeftriaxone + vancomycin DPrednisone

Ceftriaxone + vancomycin dx: acute epiglottitis Sx: rapid onset of fever, muffled voice, dysphagia, respiratory distress PE: sore throat, stridor, leaning forward, drooling, red swollen epiglottis on direct visualization Imaging: enlarged epiglottis (thumbprint sign) on lateral neck X-ray Most commonly caused by H. influenzae (decrease since Hib vaccine), Strep and Staph species Treatment is airway management and IV antibiotics (third-generation cephalosporin AND antistaphylococcal) What is the mechanism of action of vancomycin? Answer: It inhibits bacterial cell wall synthesis by blocking glycopeptide polymerization through binding tightly to D-alanyl-D-alanine portion of cell wall precursor.

A 72-year-old woman was hospitalized for atrial fibrillation with rapid ventricular rate. She is being discharged today on amiodarone for long term control of her dysrhythmia. Which of the following diagnostic studies should be followed as an outpatient?😨😨😨 AChest X-ray BCoagulation studies CComplete blood counts DRenal panels

Chest X-ray Amiodarone is a class III anti-dysrhythmic drug used to treat many common dysrhythmias Amiodarone Antidysrhythmic Multichannel blocker ACLS protocol Monitor thyroid and liver function Obtain yearly chest X-ray Eye exams Drug interactions --------------------------------------------------------------- vs Coagulation studies (B), complete blood counts (C) and renal panels (D) are not routinely monitored with amiodarone therapy; however, these studies are important for the monitoring of many different drugs. A few examples include coagulation studies for patients on warfarin, complete blood counts for patients on the antipsychotic clozapine, and renal panels for patients on angiotensin converting enzyme inhibitors. These are just a few examples.

An 8-year-old boy presents with a 2-day history of severe, watery diarrhea. He is currently living in a Haitian refugee camp. On exam, the child is afebrile but appears dehydrated. He is actively passing a large amount of watery diarrhea that is flecked with mucous. Darkfield microscopy of the stool reveals mobile organisms. Which of the following is the most likely diagnosis? ACholera BRotavirus CSalmonellosis DShigellosis

Cholera History of drinking contaminated water Severe, watery diarrhea flecked with mucus (rice water) PE will show dehydration Diagnosis is made by stool culture or dark-field microscopy Caused by toxin-producing strains of Vibrio cholerae Treatment is usually supportive: aggressive volume repletion Abx as adjunctive treatment to reduce severity and decrease sheddingDepending on resistance patterns: doxycycline, ciprofloxacin, azithromycin, tetracycline, TMP-SMX

A 14-year old girl, accompanied by her parents, presents for amenorrhea. She has never had menses. She reports no sexual activity and does not take any medications. She is a straight A student in school and plays basketball. On physical examination, her height is at the 90th percentile, weight at the 50th percentile, breasts at Tanner II, no axillary hair and no pubic hair. The rest of her examination is normal. Which of the following is the most likely diagnosis? AComplete androgen insensitivity syndrome BKlinefelter syndrome CPolycystic ovarian syndrome DTurner syndrome

Complete androgen insensitivity syndrome genetic males appear female at birth The vagina ends blindly in a pouch, and the uterus is absent. This is due to the normal production and effect of antimullerian hormone by the testes. At puberty, the diagnosis is suspected by the normal development of breasts, but menstruation does not occur and sexual hair is absent. Adult heights are commensurate with those of normal males. --------------------------------------------------------------- vs Klinefelter syndrome (B) has XXY chromosomes and presents with gynecomastia, small testicles, infertility, mild delays, and a low upper-to-lower segment ratio. Turner syndrome (D) has XO chromosomes and presents with breast buds, short stature, pedal edema, wide-spaced nipples. They may have primary amenorrhea from streak ovaries and ovarian failure.

A family brings in their 3-month-old infant who was just adopted from Honduras. They are not aware of the infant's birth history and were told the child was healthy and lived in an orphanage for the first 3 months of life. They are concerned he has significant bruising over his back. On examination, a large blue patch is present over the sacrum. Similar patches are present over the bilateral shoulders. The remainder of the examination is benign. Which of the following is the most likely diagnosis? AChild abuse BCoagulopathy CCongenital dermal melanocytosis DNevus of Ota

Congenital dermal melanocytosis

Which of the following conditions is associated with a pericardial knock on auscultation? AAcute pericarditis BConstrictive pericarditis CPericardial tamponade DRestrictive cardiomyopathy

Constrictive pericarditis Constrictive Pericarditis PE will show a pericardial knock on cardiac auscultation, Kussmaul sign, and pulsus paradoxus Exaggerated inspiratory diastolic interventricular interdependence Most commonly caused by fibrous thickening of the pericardium Treatment is pericardiectomy -------------------------------------------------------------- vs Acute pericarditis (A) is associated with a pericardial friction rub. It is best heard with the diaphragm of the stethoscope at the lower left sternal border or apex when the patient is sitting and leaning forward or in the hands-and-knees position. The classic cardiac auscultation findings of pericardial tamponade (C) include distant heart or soft heart sounds. Restrictive cardiomyopathies (D) result from systemic disorders such as amyloidosis, sarcoidosis, hemochromatosis, scleroderma, carcinoid heart disease, and endomyocardial fibrosis. Findings on physical exam depend on the stage or severity of myocardial involvement. An S3 is almost always present, and an S4 is often heard.

A 65-year-old obese man presents to your office with complaints of snoring and fatigue. His wife makes him sleep in a separate room because of his loud snoring and gasping for air in his sleep. Which of the following is the most appropriate therapy? AContinuous positive airway pressure therapy BModafinil COxygen therapy DTheophylline

Continuous positive airway pressure therapy Obstructive Sleep Apnea Risk factors: obesity, male sex, advancing age, allergies Sx: apneic episodes while sleeping, snoring, and daytime sleepiness PE: enlarged tonsils, crowded oropharynx, large neck circumference, hypertension Diagnosis is made by sleep study Treatment is weight loss, CPAP First-line treatment in children is adenotonsillectomy Adverse outcomes: CVA, motor vehicle collisions, pulmonary hypertension, cor pulmonale (right ventricular hypertrophy) -------------------------------------------------------------------- vs Modafinil (B) is a central nervous system stimulant sometimes used as adjunctive treatment of the daytime sleepiness symptoms associated with OSA. Oxygen therapy (C) may be used as an alternative in patients who are non-compliant or unable to tolerate CPAP therapy. Theophylline (D) is a phosphodiesterase enzyme inhibitor thought to stimulate the respiratory drive, however pharmacologic therapy is not effective enough to replace CPAP as first-line therapy.

A 50-year-old woman with a medical history of hypertension on lisinopril presents to the clinic with medial elbow pain for the past month. The patient reports that there was not any specific injury but reports she does intensive manual labor at work. The pain is worse when the patient plays racquetball. Vital signs today include a heart rate of 75 bpm, blood pressure of 130/80 mm Hg, respiratory rate of 20 breaths per minute, pulse oxygenation of 99% on room air, and temperature of 98.6°F. Physical examination reveals tenderness over the medial elbow and the proximal flexor aspect of the forearm. The pain is exacerbated by resisted wrist flexion. Which of the following is the most appropriate next step in the management of this patient's condition? ACounterforce brace BIntra-articular corticosteroid injection CMagnetic resonance imaging of the elbow DOral corticosteroids EX-ray of the elbow

Counterforce brace dx; Medial elbow tendinopathy (golf elbow)

A 27-year-old healthy man presents with facial pain and low-grade fever. For the last 2 days, he has felt congested and noticed green drainage from his nose. Which of the following is the most appropriate management?😨 AAntihistamines BCT scan of the sinuses CNonsteroidal anti-inflammatory drugs DSystemic antibiotics

Nonsteroidal anti-inflammatory drugs dx Acute sinusitis is defined as inflammation of the nasal cavity and paranasal sinuses lasting less than four weeks. The most common etiology of sinusitis is a viral infection with acute bacterial sinusitis responsible for only 0.5-2% of episodes. Most commonly caused by viral URI If viral, tx: supportive care Bacterial sinusitis: purulent nasal secretions and severe symptoms for ≥ 10 daysAmoxicillin-clavulanate Complications: frontal bone osteomyelitis (Pott puffy tumor), orbital cellulitis, sinus venous thrombosis, extension into meninges/brain What are three complications of bacterial sinusitis? Answer: Meningitis, orbital cellulitis, and sinus bone osteitis.

A 32-year-old woman presents to your office for her annual exam. She reveals that one month ago her 6-year-old daughter was killed in a motor vehicle accident. Since the accident she has been crying uncontrollably, can't sleep through the night, and sometimes sees her daughter walking in the upstairs hall. Which of the following is a risk factor for the development of poor bereavement outcomes? ACrying uncontrollably BDeath of a child CInsomnia DOccurrence of visual hallucinations

Death of a child Prolonged or complicated grief is characterized by persistent, disruptive emotional responses for at least six months after the death of a loved one. Risk factors include death of a child, past history of mental health issues, dependent relationship with the deceased, and poor social supports. Symptoms include difficulty with moving on and accepting the death, bitterness, numbness or detachment, agitation or being on edge, lack of trust, feelings of emptiness, and a sense that the future holds no meaning. Prolonged or complicated grief can lead to bereavement-related depression or medical sequelae such as exacerbation of chronic disease and substance abuse. Grief Reaction Stages: denial, anger, bargaining, depression, acceptance Nonpathologic symptoms: guilt, weight loss, simple hallucinations Pathologic symptoms: > 6-12 months, severe functional impairment, suicidal ideation, psychotic symptoms

A patient with irritable bowel syndrome complains mainly of lower abdominal pain. She denies diarrhea or constipation, and rarely has problematic flatulence. For this patient's abdominal pain, which of the following medications do you recommend?😨😶‍🌫️😶‍🌫️😶‍🌫️😶‍🌫️😎 ADesipramine BLoperamide CLubiprostone DRifaximin

Desipramine The pain associated with irritable bowel syndrome can be treated with selective-serotonin reuptake inhibitors like citalopram, tricyclic antidepressants like desipramine, and antispasmodics such as atropine, hyoscyamine, dicyclomine or scopolamine. Irritable Bowel Syndrome A GI syndrome characterized by chronic abdominal pain and altered bowel habits in the absence of any organic cause ------------------------------------------------------------- vs Loperamide (B), a opioid piperidine-derivative, is used to treat diarrhea, not pain, associated with irritable bowel syndrome. Lubiprostone (C) is a prostaglandin E1 derivative. Its chloride channel agonism is used in treating the constipation, not pain, associated with irritable bowel syndrome. Bloating and flatulence that are associated with irritable bowel syndrome can be treated with rifaximin (D), a semisynthetic antibiotic.

A 68-year-old woman presents to the ED with chest pain. It occurs at rest, and has been getting worse over the past 2 hours. Her past medical history includes COPD, GERD, diabetes, urinary incontinence and Factor V Leiden. Her past surgical history is significant for carpal tunnel release four years ago and total hip arthroplasty 18 months ago. Blood pressure is 168/118 mm Hg, heart rate is 100 BPM and oxygen saturation is 95% on room air. An electrocardiogram reveals ST-elevation. An initial cardiac panel shows a positive troponin level. In this scenario, which of the following in this patient is a relative contraindication to fibrinolytic therapy? ADiastolic blood pressure BFactor V Leiden CGastroesophageal reflux disease DTotal hip arthroplasty

Diastolic blood pressure Relative contraindications include systolic BP ≥ 180 or diastolic BP ≥ 110 mm Hg, as well as uncontrolled chronic hypertension, pregnancy and anticoagulant use. This patient's DBP is 118 mm Hg, above the recommended 110 mm Hg. Absolute contraindications include a history of any intracranial hemorrhage, cerebral vascular structural lesions or intracranial neoplasm. Others include ischemic stroke or head or facial trauma within the past 3 months and active bleeding or bleeding diathesis. ---------------------------------------------------------------- vs Bleeding diatheses (tendency or predisposition), such as von Willebrand's disease and Factor V deficiency, are absolute contraindications to fibrinolytics. Factor V Leiden (B) is a pro-thrombotic coagulopathy, not a bleeding diathesis. As such, this is not a contraindication. Active peptic ulcer disease, not GERD (C), is a relative contraindication to fibrinolytic therapy. Major surgery, such as total hip arthroplasty (D), is a relative contraindication if it occurred within 3 weeks of acute coronary symptom onset.

A 7-year-old boy presents to the emergency department with cough and chest pain. The cough has been present for several days, but the chest pain has been present for only one day. His vital signs are notable for a temperature of 38.8°C, heart rate of 150, blood pressure of 78/49, respiratory rate of 50, and oxygen saturation of 94%. He is ill appearing, has coarse breath sounds on lung auscultation, a systolic murmur and a gallop on his cardiac exam, and his liver is palpable 4 centimeters below the right costal margin. His pulses are palpable and capillary refill is 5 seconds. Which of the following is the most likely complication? AAortic valve insufficiency BConstrictive pericarditis CDilated cardiomyopathy DLeft ventricular outflow tract obstruction

Dilated cardiomyopathy Myocarditis Patient presents with fatigue, fever, chest discomfort, dyspnea, and palpitations PE will show tachycardia disproportionate to fever or discomfort Echocardiogram will show decreased ventricular ejection fraction with hypokinesis and wall motion abnormalities Gold standard for diagnosis is an endomyocardial biopsy

A 25-year-old woman presents to her obstetrician for evaluation of recurrent second trimester pregnancy loss. During each of her previous two pregnancies, she presented during week 15 with painless cervical dilation and effacement with subsequent rapid delivery of a preterm infant. Which of the following is a risk factor for this woman's cervical insufficiency? AEhlers-Danlos syndrome BHistory of pelvic inflammatory disease CHuman papillomavirus infection DObesity

Ehlers-Danlos syndrome congenital risk factor for cervical insufficiency. Cervical insufficiency is defined by the American College of Obstetricians and Gynecologists (ACOG) as the inability of the uterine cervix to retain a pregnancy throughout the second trimester, in the absence of uterine contractions The diagnosis of cervical insufficiency is usually based on history of recurrent midtrimester loss, risk factors, and a transvaginal ultrasound measurement of cervical length. Cerclage placement is recommended at 12-14 weeks for with a history suggestive of cervical insufficiency. What is a contraindication for cerclage placement? Answer: Rupture of membranes. -------------------------------------------------------------- vs A history of pelvic inflammatory disease (B) is not a risk factor for cervical insufficiency. Human papillomavirus infection (C) is not an independent risk factor for cervical insufficiency. Obesity (D) is not a risk factor for cervical insufficiency.

A 19-year-old G1P0 woman at 26 weeks presents with abdominal pain after being involved in a motor vehicle collision. External pelvic examination reveals vaginal bleeding. Which of the following is true regarding this presentation? AA normal ultrasound rules out placental abruption BDisseminated intravascular coagulation is uncommon in placental abruption CEarly pelvic digital examination should be performed DEmergent fetal monitoring and obstetric consultation are required

Emergent fetal monitoring and obstetric consultation are required

A 63-year-old man presents with dizziness. He states that when he turns his head to the right, he feels an intense sensation of the room spinning, with nausea and vomiting. The symptoms resolve in minutes with rest. His physical examination is remarkable for right-beating nystagmus when his head is turned to the right but is otherwise normal. What management is indicated? AAntibiotics BEpley maneuver CNoncontrast head CT DPrednisone When backing out of her garage, a patient complains of "the car is spinning" when she turns her head quickly to look behind her. It lasts about half-a-minute, but does not occur when she looks behind the car through the rear-view mirror. Which of the following is most likely to be found during a physical examination? AAllodynia BDecreased proprioception CPositive Dix-Hallpike maneuver DPositive Epley maneuver

Epley maneuver Benign paroxysmal positional vertigo (BPPV) Patient presents with sudden onset of sensation of room spinning in connection with positional changes of the head, lasting seconds to minutes Diagnosis is made by Dix-Hallpike Most commonly caused by the presence of an otolith in the labyrinth system Treatment is Epley maneuver

Which of the following medications blocks the release of stored thyroid hormone? AIodine BMethimazole CPropranolol DPropylthiouracil name an alternative if patient is allergic to said medication

Iodine It should only be given one hour after either propylthiouracil or methimazole, both of which block synthesis of thyroid hormone, because an iodine load can increase synthesis of the hormone. Lithium is an alternative medication that can block release of thyroid hormone in iodine-allergic patients. --------------------------------------------------------------- vs Methimazole (B) is a thionamide which inhibits oxidation and binding of iodine to thyroglobulin, thereby decreasing synthesis of thyroid hormone. It is given via oral or nasogastric route. Propranolol (C) is the beta-blocker of choice in managing the adrenergic response found in thyroid storm. It also blocks conversion of T4 to T3. It can be given intravenously or orally, depending on the severity of symptoms. Propylthiouracil (D) is also a thionamide which blocks synthesis of thyroid hormone similar to methimazole. Unlike methimazole, it blocks conversion of T4 to T3 as well. The usual dose of propylthiouracil is 600-1000 mg loading oral dose followed by 300 mg every 6 hours.

A 32-year-old man who works as a landscaper presents to the clinic after feeling that something blew into his eye while he was working. He now has a foreign body sensation in his left eye and tearing. Examination with a slit lamp does not reveal an obvious foreign body. Which of the following is the most appropriate next step in managing this patient? AApplication of a topical nonsteroidal anti-inflammatory drug BEmergent referral to ophthalmology CExamination with fluorescein under a cobalt blue light DMeasurement of the intraocular pressure using tonometry

Examination with fluorescein under a cobalt blue light ---------------------------------------------------------------- vs Application of a topical nonsteroidal anti-inflammatory drug (A) should be considered as a treatment option after diagnosis is confirmed. Emergent referral to ophthalmology (B) is not necessary with uncomplicated corneal abrasions. Measurement of the intraocular pressure using tonometry (D) may be an important part of the physical exam in a patient with an eye complaint, however, is not the next best step in managing this patient with a suspected corneal abrasion.

A 15-year-old girl presents to urgent care with a nonproductive cough and low-grade fever that developed insidiously over the previous week. Physical exam reveals diffuse rales on pulmonary auscultation, and chest X-ray shows diffuse non-focal infiltrates. Which of the following is the most accurate statement regarding this patient's condition? AExtrapulmonary manifestations may include hemolysis and CNS involvement BMycoplasma pneumoniae is the most common etiology in all age groups CPenicillin is the most appropriate treatment DThe formation of cold agglutinins is a positive confirmatory test

Extrapulmonary manifestations may include hemolysis and CNS involvement Atypical bacterial infections are also more likely to present with extrapulmonary manifestations such as skin rashes, hemolysis (secondary to the formation of cold agglutinin antibodies), CNS involvement (aseptic meningitis, cerebellar ataxia, cranial nerve palsies), and gastrointestinal symptoms. However, these typical and atypical features frequently overlap and cannot be used consistently to distinguish between different etiologic agents. Atypical Pneumonia Patient presents with the gradual onset of dry cough, dyspnea, and extrapulmonary symptoms such as headache, myalgias, fatigue, and GI disturbance PE: rales with auscultation of lung fields Chest X-ray: interstitial infiltrate Most commonly caused by Mycoplasma pneumoniae: young Legionella: smokers, aerosolized water, air travel, GI symptoms, hyponatremia Chlamydophila pneumoniae: close quarters outbreaks, young, follows pharyngitis Coxiella burnetii: livestock exposure, include LFTs Chlamydophila psittaci: bird exposure, hyperpyrexia, severe HA Treatment: either empiric therapy (covering typical and atypical PNA) or directed therapy (azithromycin for C. pneumoniae)

Which of the following best describes the most common symptoms of acute human immunodeficiency virus infection? ADiarrhea and abdominal pain BFever, malaise, myalgias, sore throat, and rash CFever, productive cough, and shortness of breath DMost individuals are asymptomatic

Fever, malaise, myalgias, sore throat, and rash

A 72-year-old man has been more fatigued lately. As a result, his primary practitioner orders an electrocardiogram. A P wave is present for every QRS complex and the PR interval is 225 milliseconds in length. Which of the following is the diagnosis? AFirst degree atrioventricular block BHigh-grade atrioventricular block CMobitz type I atrioventricular block DThird degree atrioventricular block

First degree atrioventricular block First‐Degree Heart Block Associated with aging, digitalis, ischemia, inflammation, cardiomyopathies Rhythm will be regular PR interval will be > 0.20 s (200 msec) and constant

An 18-year-old man is diagnosed with condyloma acuminatum in the genital area. Which of the following physical exam findings is most consistent with this diagnosis? AFlesh-colored, exophytic lesions BPainless ulceration CShallow, painful ulcerations DUmbilicated, dome-shaped lesions

Flesh-colored, exophytic lesions Condyloma Acuminata Patient presents with genital lesions PE will show cauliflower-like lesion Most commonly caused by HPV 6 & 11 HPV 16 & 18 most commonly associated with squamous cell carcinoma Most common STI Patient-applied vs clinician-applied treatments

An 82-year-old man with a history of hypertension, atrial fibrillation, and deep vein thrombosis presents to the emergency department for weakness, fatigue, and dark, tarry stools for the past several days. His current medications include amlodipine, metoprolol, and warfarin. Vital signs reveal a temperature of 36.9°C, blood pressure of 91/59 mm Hg, heart rate of 115 bpm, respiratory rate of 17/min, and SpO2 of 98% on room air. Physical exam reveals an acutely ill-appearing man with an abdomen that is soft, nontender, and nondistended to palpation in all four quadrants. Stool guaiac exam is positive for occult blood. The remainder of his physical exam is normal. Labs reveal WBC 5,000/µL, hemoglobin 8 g/dL, platelets 200,000/µL, INR 8, and fibrinogen 200 mg/dL. Prothrombin complex concentrate is not available for administration. Which of the following is the best next immediate treatment for this patient? AAndexanet alfa BCryoprecipitate CFresh frozen plasma DIdarucizumab EProtamine sulfate

Fresh frozen plasma dx: warfarin-associated upper gastrointestinal (GI) bleeding ------------------------------------------------------------- vs Andexanet alfa (A) is a reversal agent for factor Xa inhibitors such as apixaban or rivaroxaban. This would have no effect on bleeding associated with warfarin. Cryoprecipitate (B) is indicated in the treatment of disseminated intravascular coagulation (DIC) to aid in the reversal of low fibrinogen levels. This patient does not meet the criteria for DIC as his fibrinogen is within normal limits. Idarucizumab (D) is used to reverse the effects of dabigatran, which is a direct thrombin inhibitor. This is not indicated in the treatment of warfarin-related bleeding or warfarin reversal. Protamine (E) is used to reverse the effects of heparin and is not indicated in the treatment of warfarin-related bleeding or warfarin reversal.

A 28-year-old man presents to the outpatient clinic after injuring his right ankle earlier today. He was playing soccer and describes planting his right foot on the turf and inverting the foot. He was not able to continue playing but walked off the field with assistance. He applied ice to the ankle but noted a large amount of swelling by the time he got home. He is a graduate student and plays sports recreationally. He reports no previous injuries to the ankle. On physical examination, he walks with an antalgic limp on the right. Diffuse swelling is noted over the right ankle, more on the lateral side, and tenderness over the anterior talofibular ligament. There is mild laxity to anterior drawer testing on the right compared with the left side. Subtalar tilt testing is limited by pain. X-rays of the ankle show soft tissue swelling and no bony abnormalities. In addition to elevation, ice, and over the counter pain medication which of the following is the most appropriate initial treatment? AActivity as tolerated BFunctional ankle brace and gradual weight-bearing CImmobilization in a short leg cast or boot for 4 weeks DNon-weight-bearing with crutches for 6 weeks ESurgical reconstruction o

Functional ankle brace and gradual weight-bearing grade 2 injury of the anterior talofibular ligament, the most common ligament involved in an inversion ankle injury, which is also the most common mechanism in ankle sprains. A sprain is a partial or complete tear of a ligament. Sprains are graded from 1 to 3 based on the degree of injury to the ligament. The Ottawa ruleswere developed to screen patients for the need for imaging in the acute setting. Indications for X-ray imaging include pain and tenderness to palpation over the medial or lateral malleoli, midfoot, fifth metatarsal, or navicular and inability to bear weight immediately after the injury or for four steps in the clinic. Treatment of ankle sprains involves reducing swelling through ice, compression, and elevation and pain relief with NSAIDs. A functional brace is useful to support the ankle and allow for early weight-bearing. Patients can use crutches initially and gradually increase weight-bearing as tolerated. A grade 1 sprain involves stretching of some of the ligament fibers, and there is no instability to stress on physical examination. Grade 2 sprains involve tearing of some of the fibers and result in increased laxity of the joint but with a firm endpoint. Grade 3 sprains are complete tears of the ligament with more severe laxity on exam and a soft endpoint to stress testing. What is a high ankle sprain? Answer: An injury to the tibiofibular (syndesmotic) ligament, usually due to an external rotation mechanism. ------------------------------------------------------------------- vs Activity as tolerated (A) in addition to elevation, ice, and over the counter medications are appropriate for a mild or grade 1 sprain, but a grade 2 sprain is more appropriately treated with a brace and initial limited weight-bearing. Immobilization in a short leg cast or boot for 4 weeks (C) is not appropriate. A short period (7 to 10 days) of immobilization may be appropriate for a severe sprain to allow resolution of pain and swelling. Non-weight-bearing with crutches for 6 weeks (D) is not appropriate or necessary. The patient can begin weight bearing as pain allows. Surgical reconstruction of the lateral ankle ligaments (E) is rarely indicated in

Which of the following treatments decreases the need for repeat medical visits for children with croup? ACodeine BCool mist therapy CGlucocorticoids DNebulized racemic epinephrine

Glucocorticoids Laryngotracheitis (Croup) Patient will be a nontoxic-appearing child, 6 months to 3 years old URI symptoms with barky seal-like cough, inspiratory stridor, low-grade fever X-ray will show steeple sign on PA view Most commonly caused by parainfluenza virus Treatment is steroids, racemic epinephrine -------------------------------------------------------- vs Nebulized racemic epinephrine (D) is used as rescue treatment for children with moderate to severe croup but has not been shown to have an impact on the patient's respiratory status more than 2 hours after administration.

A 42-year-old man displays personality changes and confusion for 2 days. He reports no pain. Upon presentation, you discover a weak right leg, speech impairment, and low-grade fever. Upper extremity and left leg strength, rectal sensation, muscle tone, reflex testing, and four-extremity sensory examination are normal. Nuchal rigidity, involuntary tremors, clonus, and spasticity are absent. A brain MRI shows left temporal lobe edema. Cerebrospinal fluid analysis has an elevated number of red blood cells and the Gram stain is negative for bacteria. Which of the following is the most likely diagnosis? AEncephalitis BMeningitis CMyelitis DSydenham chorea

Herpes Encephalitis Patient presents with HA, fever, behavioral changes CSF will show ↑ RBCs MRI will show temporal lobe edema Treatment is acyclovir Most common cause of nonepidemic encephalitis in the US

Perforation is a complication that occurs in 10% of patients with acute cholecystitis and is characterized by which of the following? ACoarse cough BHypoactive bowel sounds CJaundice DNausea and vomiting

Hypoactive bowel sounds Other symptoms include high fever, systemic signs of toxicity (tachycardia and increased respiratory rate), and increased abdominal pain with rebound tenderness.

An 8-year-old male presents to the clinic with left ear pain and drainage that began three days ago. Physical exam reveals an erythematous and mildly edematous left ear canal with brown debris. The visualized portion of the tympanic membrane is intact. There is pain with manipulation of the auricle. Which of the following is the most appropriate next step in the management of this patient? AIrrigate the ear canal to remove debris BPlace a wick in the ear canal CStart oral antibiotics DStart topical antibiotics

Irrigate the ear canal to remove debris dx: External otitis History of swimming or moisture exposure Malodorous discharge and pruritus PE will show pain with palpation of tragus or pinna Most commonly caused by Pseudomonas aeruginosa Treatment is topical antimicrobials with or without steroids Necrotizing otitis externa: a complication seen in those with diabetes or immunocompromise ------------------------------------------------------------------- vs Placement of an ear wick (B) is done when there is severe swelling of the ear canal to facilitate the entry of topical medications. Oral antibiotics (C) are reserved for severe cases of external otitis that do not resolve with topical medications. Topical antibiotics (D) should be initiated after the debris has been removed.

A 44-year-old woman presents with a bilateral throbbing headache and nausea that was preceded by vision changes of flashing lights and blind spots. Her past medical history is significant for poorly controlled hypertension. Currently, her blood pressure is 182/100 and neurological examination is normal. CT scanning reveals no intracranial pathology. Which of the following is the most appropriate headache treatment at this time? AHigh flow oxygen BIbuprofen CSumatriptan alone DSumatriptan plus ergotamine

Ibuprofen dx:Migraine Headache Risk factors: female sex, family history Gradual onset, unilateral > bilateral, throbbing, pulsating headache Without aura: most common, nausea or vomiting, photophobia, phonophobia Aura: scotoma, flashing lights, sounds Diagnosis is made clinically Treatment Abortive Rx: triptans, DHE, antiemetics, NSAIDs Prophylaxis: TCAs, beta-blockers, anticonvulsants (valproic acid, topiramate), CCBs Triptans, DHE: avoid in uncontrolled HTN or CV disease ". Triptan medications (C and D) have adrenergic-agonistic properties. Therefore, they should not be used in patients with cardiovascular or cerebrovascular disease, angina, severe hypertension and peripheral vascular disease. Clinicians must never combine ergot-derivatives with triptan medications. " ------------------------------------------------------------- vs High flow oxygen (A) is used in treating acute cluster headache and not migraine. This scenario does not match the description of cluster headaches. Triptan medications (C and D) have adrenergic-agonistic properties. Therefore, they should not be used in patients with cardiovascular or cerebrovascular disease, angina, severe hypertension and peripheral vascular disease. Clinicians must never combine ergot-derivatives with triptan medications.

A 23-year-old woman presents with intermittent postcoital vaginal bleeding and persistent purulent vaginal discharge for one week. On exam, her cervix is erythematous and friable. There is no cervical motion tenderness on exam. Which of the following is the best next step? AAdmit the patient for parenteral antibiotic therapy BInitiate empiric antibiotic therapy CObtain cervical culture DObtain vaginal discharge swab for nucleic acid amplification testing and follow up results with primary care physician

Initiate empiric antibiotic therapy Chlamydia Cervicitis Diagnosis is made by nucleic acid amplification testing (NAAT) Most commonly caused by Chlamydia trachomatis Treatment is doxycycline (100 mg BID x 7 days), azithromycin should be used in pregnancy Reinfection testing after treatment:

A 34-year-old woman presents with lower abdominal pain and fever. She had a full-term delivery via cesarean section 4 days ago. On examination, she has a temperature 38.6°C and a heart rate of 112 beats/minute. Her lower abdominal incision is intact with no fluctuance, surrounding erythema, or drainage. She has midline lower abdominal tenderness to palpation. On pelvic examination, she has uterine tenderness with foul smelling lochia. What is the most appropriate management of this patient? AIntravenous clindamycin and gentamicin BIntravenous vancomycin COral cephalexin DOral metronidazole

Intravenous clindamycin and gentamicin dx: Postpartum endometritis Endometritis Most common postpartum infection Risk factors: C-section, prolonged labor, prolonged ROM, chorioamnionitis, meconium-stained amniotic fluid, maternal DM, GBS colonization Etiology: polymicrobial (usually two to three aerobic and anaerobic species) Early-onset disease (< 48 hours after delivery) or fever > 101.3 °F Suspect Streptococcus pyogenes Sx: fever, abdominal pain, foul-smelling lochia PE: uterine tenderness and purulent drainage from the uterus Labs: leukocytosis Treatment is clindamycin + gentamicin GBS colonized: add ampicillin or use ampicillin-sulbactam

58-year-old man presents with 3 months of headache and transient diplopia. He also reports that chewing tough foods has been progressively painful. Laboratory testing reveals a normocytic anemia and ESR of 88 mm/hour. A brain CT is normal. Biopsy reveals fragmented elastic lamina, CD4+ lymphocytes and macrophages. Which of the following is the best next step to manage this patient's condition? AIntravenous methylprednisolone BOral methotrexate COral prednisone DTemporal artery resection

Intravenous methylprednisolone dx Giant cell arteritis, as known as temporal arteritis

A 57-year-old man with a history of chronic kidney disease (baseline creatinine of 3.3 mg/dL) and liver cirrhosis presents with confusion. Examination reveals scleral icterus and asterixis. Vital signs and serum glucose are normal. What management is indicated? AHead CT and lumbar puncture BLactulose CNeomycin DProtein-restricted diet

Lactulose dx: Hepatic Encephalopathy History of infection, dehydration, medications, GI bleed, hypoxia, hypoglycemia Presents with confusion PE will show scleral icterus and asterixis Most commonly caused by toxin accumulation due to liver dysfunction Treatment is lactulose, rifaximin CSF glutamine correlates with severity

A 45-year-old man comes to the office with a chief complaint of ringing in his right ear with decreased hearing on the right side. He does not recall any trauma. On exam there is no cerumen impaction or carotid bruits. Audiographic evaluation confirms right-sided hearing loss only. What is the next step in evaluation and treatment? AAudiology appointment for hearing aid BCarotid doppler of the neck CMagnetic resonance imaging of the internal auditory canal DNasolaryngoscopy

Magnetic resonance imaging of the internal auditory canal If tinnitus is accompanied with hearing loss that is asymmetric or unilateral, this requires further evaluation with magnetic resonance imaging of the internal auditory canal to rule out acoustic neuroma.

You are examining a newborn male. Scrotal palpation reveals the absence of a testicle. If the condition does not spontaneously resolve, which of the following is the most appropriate intervention? AOophorectomy BOrchiopexy CPreputioplasty DSalpingectomy

Orchiopexy Cryptorchidism History of low birth weight and prematurity PE will show an empty or hypoplastic scrotal sac Treatment is surgical orchiopexy Complications include testicular torsion, infertility, and malignancy

A 45-year-old woman with no significant medical history presents to the clinic with a gradual onset of muscle weakness that has developed over the past 3 months. The patient reports minimal muscle pain and reports no muscle stiffness. Vital signs today include a heart rate of 65 bpm, blood pressure of 125/82 mm Hg, respiratory rate of 20 breaths per minute, pulse oxygenation of 99% on room air, and temperature of 98.6°F. Physical examination reveals symmetric 4/5 muscle strength with shoulder or hip movements and intact 5/5 distal muscle strength. You also note symmetric erythematous papular lesions on the bilateral metacarpophalangeal and interphalangeal joints, which the patient reports started about 6 months ago. You assess laboratory studies, and the results are notable for a creatine kinase of 10,500 U/L. Which of the following conditions is known to be associated with the suspected diagnosis?😨😳👀👀 AAortic aneurysm BHyperlipidemia CMalignancy DPulmonary embolism ESudden vision loss

Malignancy Dermatomyositis Risk factors: female sex, age 40-60, occult malignancy Sx: insidious, painless, proximal muscle weakness (polymyositis) and a rash, dysphagia PE: heliotrope rash, Gottron papules, mechanic's hands, photodistributed poikiloderma (shawl sign, v-sign) Labs: ↑ CK and aldolase Dx: MRI, EMG, muscle biopsy Tx: steroids, methotrexate or azathioprine Antisynthetase syndrome: anti-Jo-1 antibodies, interstitial lung disease, worse prognosis Age-appropriate screening for malignancy What is the most common pulmonary manifestation of dermatomyositis? Answer: Interstitial lung disease.

An 8-year-old boy presents with his parents to primary care due to frequent maroon colored stools for the past few weeks. He is otherwise asymptomatic and reports no history of abdominal pain or diarrhea. He has a history of occasional constipation as an infant but is otherwise healthy. He takes no medications. His vitals are pulse 85 bpm, temperature 99.3°F, BP 90/60 mm Hg, and RR 18 breaths/minute. Physical exam reveals a soft, nontender abdomen with no guarding or rebound tenderness. Labs drawn last week reveal mild microcytic anemia, and his upper and lower endoscopy did not show any abnormality. Which of the following will be the most useful in confirming the most likely diagnosis? ABacterial stool culture BCT scan of the abdomen/pelvis CElimination of dairy products DMeckel scan EPlain films of the abdomen

Meckel scan Clinicians should be suspicious for Meckel diverticulum in patients with evidence of painless GI bleeding, evidence of aGI bleed with normal endoscopy results, and evidence of a GI bleed and abdominal pain with no other clear etiology. Meckel scan is the diagnostic of choice for Meckel diverticulum for stable patients who have a history and physical exam suspicious for the disease. Treatment of patients who are symptomatic usually consists of resecting the diverticulum.

Which of the following factors indicates a need for esophagogastroduodenoscopy in a patient with dyspepsia? AInadequate response to calcium carbonate therapy BMale sex CMelena DWorsened symptoms at night

Melena History and exam should focus on identifying alarm features of gastroesophageal malignancy, such as unintentional weight loss, progressive dysphagia,odynophagia, persistent vomiting, gross gastrointestinal bleeding, palpable mass or lymphadenopathy, family history of upper gastrointestinal malignancy, and unexplained iron-deficiency anemia. The need for esophagogastroduodenoscopy (EGD) depends on a patient's age and clinical presentation. EGD should be performed when patients < 60 years of age present with clinically significant weight loss, rapidly progressive alarm features, overt gastrointestinal bleeding, or if more than one alarm feature is present. EGD should be performed on all patients ≥ 60 years of age with new dyspepsia. ( look at the image )

A 61-year-old man presents with loss of appetite, unintentional weight loss, and dyspepsia. Endoscopy shows a gastric mass at the curvature of the stomach, and he is diagnosed with gastric carcinoma. Which of the following is true about his condition? AChemotherapy and radiation are standard initial therapy for localized disease BMen have a higher risk of developing this condition CThe 5-year survival rate of advanced disease is 80% DWhite individuals are more likely to develop this condition

Men have a higher risk of developing this condition Gastric Carcinoma Risk factors: male sex, history of H. pylori infection Sx: loss of appetite, unintentional weight loss PE: supraclavicular node (Virchow node), left axillary node (Irish node), periumbilical node (Sister Mary Joseph node) Adenocarcinoma is most common Patients may report abdominal pain, loss of appetite, nausea, difficulty swallowing, and unintentional weight loss as the tumor advances. Iron deficiency anemia and occult blood in the stool may be found A gastric mass may be palpable in some patients. Endoscopy with cytology and biopsy are recommended for any patient over 50 years who requires ongoing acid suppression to control symptoms of dyspepsia to promote early detection of gastric carcinoma.

A 78-year-old man undergoes laparoscopic surgery for a small bowel obstruction. Which metabolic disturbance is he most at risk for postoperatively? AMetabolic acidosis BMetabolic alkalosis CRespiratory acidosis DRespiratory alkalosis

Metabolic alkalosis

A 5-year-old boy is brought to the emergency department after being found unresponsive at home. He was found lying on the floor in his parent's room with prescription medications scattered all over. His parent called 911, and he was immediately rushed to the hospital by ambulance. On examination, the boy is sedated with a respiratory rate of 15/min, blood pressure of 70/50 mm Hg, oxygen saturation of 92% on room air, pupils 1-2 mm and reactive to light, and 1+ reflexes on all extremities. His blood sugar is 200 mg/dL. An ECG reveals QTc interval prolongation, followed by polymorphic ventricular tachycardia with a rate of 190 bpm and QRS complexes twisting around the isoelectric line. Which of the following is the most likely medication ingested? AClonidine BMethadone CPropranolol DSalicylate

Methadone dx: opioid ingestion opioid toxidrome of respiratory depression, sedation, and miosis. Signs of more severe toxicity can includehyporeflexia, hypotension, bradycardia, and hypothermia. Methadone is particularly associated with a prolonged QTc interval and risk of torsades de pointes, -------------------------------------------------------------------- vs Clonidine (A) toxicity manifests as lethargy, miosis, and bradycardia. Although findings may be similar to opioid overdose, QTc interval prolongation and torsades de pointes are not seen with clonidine. In fact, clonidine has been shown to decrease the QT interval. Propanolol (C) toxicity causes bradycardia and hypotension that typically develops within six hours of ingestion. It is one of the beta-blockers with significant membrane-stabilizing activity when taken at toxic doses, which can result in a widened QRS interval and dysrhythmias such as heart block. Seizures, delirium, and hypoglycemia may also be seen. Clinical manifestations of salicylate (D) toxicity include nausea, vomiting, diaphoresis, and tinnitus. Moderate cases can manifest with tachypnea, tachycardia, and altered mental status.

A 77-year-old woman presents with dizziness and being light headed. She denies any new medications or other recent changes. Vitals are temperature 98.0°F, blood pressure 93/56 mm Hg, and pulse 72 bpm. Physical exam is unremarkable. Her medications include metoprolol 50 mg daily, atorvastatin 10 mg daily, vitamin D 1,000 IU daily, and acetaminophen 500 mg as needed. Which of the following is the most likely cause of her dizziness? AAcetaminophen use BAtorvastatin use CMetoprolol use DVitamin D supplementation

Metoprolol use ------------------------------------------------------------------ vs Acetaminophen use (A) can cause light headedness, along with nausea, vomiting, loss of appetite and a host of other common side effects. It would not likely cause this patient's hypotension. Atorvastatin use (B) can cause weakness, confusion, fever, unusual tiredness, dark colored urine and many other side effects, but not hypotension. Vitamin D (D) does not usually cause side effects especially at this dose. Taking doses higher than 4,000 to 5,000 IU daily can cause weakness, fatigue, headache, tinnitus, metallic taste, and nausea, but not hypotension.

A 31-year-old man presents to his primary care physician with fevers, chills, and night sweats. After a physical exam and a series of laboratory tests, the patient undergoes a lymph node biopsy that reveals Reed-Sternberg cells. Three days later, while awaiting treatment, he presents to his physician complaining of generalized swelling and is found to have 4+ protein in his urine. Which of the following pathological findings is most likely to be found on renal biopsy in this patient? AAmyloidosis BFocal segmental glomerulosclerosis CMinimal change disease DRapidly progressive glomerulonephritis

Minimal change disease Hodgkin's lymphoma complicated by minimal change disease. Reed-Sternberg cells on biopsy are indicative of Hodgkin's lymphoma. ----------------------------------------------------------------- vs Rapidly progressive glomerulonephritis (D) is characterized by crescent moon-shaped glomeruli and is associated with a rapid decrease in GFR. Goodpasture syndrome is associated with this pathology. Amyloidosis of the kidney (A)is characterized by amyloid protein deposits within the glomeruli. FSGS (B) is the most common nephrotic syndrome in adults and is characterized by focal (only some glomeruli affected) and segmental (only part of each affected glomerulus is damaged) pathology. However it is not highly associated with Hodgkin's lymphoma.

A 29-year-old woman presents to the emergency department with a complaint of bloody diarrhea and abdominal cramping. Recently, she ate a rare hamburger at a birthday party for her 4-year-old son. He ate hot dogs instead, and has not been ill. A stool specimen is positive for Escherichia coli O:157. Which one of the following should you do next? AMonitor blood urea nitrogen/creatinine levels BProvide levofloxacin prophylaxis to her close contacts CRenal biopsy DRenal ultrasound

Monitor blood urea nitrogen/creatinine levels Hemolytic Uremic Syndrome (HUS) Labs will show anemia, renal failure, thrombocytopenia (ART) Most commonly caused by E. coli O157:H7, also S. pneumoniae, HIV, complement deficiency Tx: supportive Avoid Abx if suspect E. coli

A 62-year-old man presents to the urgent care clinic reporting 2 hours of right lower leg and foot pain. He smokes one pack of cigarettes per day and has done so for 20 years. He takes lisinopril for high blood pressure and states he has high cholesterol but has not been taking his cholesterol medication due to undesirable side effects. He reports no history of diabetes mellitus. His vital signs are a blood pressure of 150/90 mm Hg, heart rate of 88 bpm, respirations of 18/minute, and temperature of 98.8°F. On physical exam, his right foot is pale and cool to the touch with no palpable pedal pulsations. Which of the following history or physical exam findings is consistent with the most likely diagnosis? ADependent rubor of the right foot BGradual onset of symptoms CIndurated superficial veins of the right leg DMuscle weakness of the right leg ENonhealing lower extremity ulcer

Muscle weakness of the right leg dx: Acute limb ischemia is defined as sudden loss of limb perfusion compromising tissue viability and is most often due to arterial thromboembolism six Ps: pain, pallor, poikilothermia, paresthesias, paralysis, and pulselessness. Poikilothermia refers to coolness of the involved extremity, and paralysis refers to muscle weakness of the involved muscle groups due to lack of blood flow. Acute limb ischemia is a medical emergency and must be addressed within a few hours of symptom onset to save the involved limb. Treatment of acute limb ischemia involves reperfusion through surgical open embolectomy

A 14-year-old girl has been diagnosed with primary amenorrhea. Pelvic ultrasound reveals an absent uterus. Genetic testing confirms a 46,XX karyotype. Which of the following is the most likely diagnosis? AAsherman syndrome BImperforate hymen CMüllerian dysgenesis DTurner syndrome

Müllerian dysgenesis ----------------------------------------------------------- vs Asherman syndrome (A), past uterine procedures, like cesarean section or dilation and curettage, cause formation of uterine synechiae. Vaginal abnormalities include vaginal agenesis, in which the vagina does not develop; transverse vaginal septum, in which there is a failed fusion of the portions of the vagina; and imperforate hymen (B), in which there is a persistence of the urogenital membrane. The most common ovarian cause of amenorrhea is primary ovarian failure due to gonadal dysgenesis or agenesis. The abnormal ovaries in these patients do not produce estrogens, which results in increased gonadotropin levels. This condition is known as hypergonadotropic hypogonadism. Amenorrhea caused by gonadal dysgenesis or agenesis is usually due to sex chromosome abnormalities, such as Turner syndrome (45,XO) (D), which cause a premature depletion of primordial follicles and subsequent estrogen deficiency.

A 27-year-old woman has pleuritic chest pain and mild shortness of breath. A D-dimer was performed and is negative. What value best estimates the likelihood that this patient does not have a pulmonary embolism given the negative test result? ANegative predictive value BPositive predictive value CSensitivity DSpecificity

Negative predictive value ikelihood that a person with a negative test does not have the disease in question.

A 3-year-old boy is brought to the emergency department by his parent because of possible poisoning. He was found in the garage with furniture polish on his cheeks, mouth, and clothes. The parent saw him coughing but reports no choking or vomiting. On examination, the boy is active with normal vital signs. Which of the following is the next best step? AConsult pulmonology BGive activated charcoal CObtain chest X-ray DPerform gastric lavage

Obtain chest X-ray Toxic Effects of Gases, Fumes, Vapors Ammonia: cleaning product odor, fertilizers Chlorine: swimming pool odor Hydrocarbons: fuels Hydrogen sulfide: rotten egg odor, cyanide-like effect Nitrogen oxides: combustion Phosgene: hay odor, plastics and textiles Pulmonary edema --------------------------------------------------- vs pulmonology (A). Giving activated charcoal (B) is not useful because it does not bind the common hydrocarbons and can also induce vomiting. Performing gastric lavage (D) is contraindicated, given the risk of aspiration.

A 16-year-old girl presents to your clinic with decreased energy, poor performance in school, and constipation for the past eight months. Her physical exam is unremarkable, but you are concerned she may have thyroid disease. Which of the following is the most appropriate next best step in management? AObtain a complete blood count with differential BObtain an ultrasound of the thyroid CObtain free triiodothyronine (FT3) levels DObtain thyroid-stimulating hormone (TSH) and free thyroxine (FT4) levels

Obtain thyroid-stimulating hormone (TSH) and free thyroxine (FT4) levels Hyperthyroidism Sx: heat intolerance, palpitations, weight loss, tachycardia, and anxiety PE: hyperreflexia, goiter, exophthalmos, pretibial edema Labs: low TSH and high free T4 Most commonly caused by Graves disease (autoimmune against TSH receptor) Tx: methimazole or PTU PTU in the first trimester of pregnancy

A 68-year-old man presents with low back pain for the last several weeks. His pain is worse after walking. After several minutes of walking, he has to sit down due to the pain. The pain eventually dissipates following rest. Extension of his lumbar spine causes increased pain that radiates down his bilateral lower extremities. What is the next best step in managing this patient? AOrder magnetic resonance imaging of the lumbar spine BPerform a facet joint corticosteroid injection CPrescribe oral prednisone and analgesics DRefer to orthopedic surgery

Order magnetic resonance imaging of the lumbar spine Spinal Stenosis Risk factors: age over 60 Low back pain and stiffness when walking that is relieved when leaning forward Diagnosis is made by MRI Most commonly caused by narrowing of the lumbar spinal canal with compression of the nerve roots Management includes physical therapy, steroid injections (lack of evidence of benefit), surgery

An 8-year-old girl presents with her father to the clinic because of itchy bumps on her legs for several weeks. Yesterday, there were a few bumps on the girl's legs, which she has been constantly scratching. She did recall that she had insect bites on her legs two months ago when she went hiking with her brother and father although she has not had any insect bite recently. Vital signs include T of 98.8°F, BP 104/66 mm Hg, HR 92 bpm, RR of 16/minute, and SpO2 of 98% on room air.On skin examination, there are multiple 0.5 cm to 1 cm papules with central crusts and excoriations on both her legs. Which of the following is the most likely diagnosis?😨😨 AAtopic dermatitis BEhrlichiosis CLangerhans cell histiocytosis DPapular urticaria EScabies

Papular urticaria hypersensitivity disorder in which insect bites, most often those of fleas, mosquitoes, or bedbugs, lead to recurrent and sometimes chronic itchy papules on exposed areas of skin (like the arms, lower legs, upper back, and scalp). Papular urticaria is reported predominantly in young children (typically two to 10 years of age).

Which of the following medications is first-line therapy to treat Parkinson disease? AAmantadine BLevodopa CPropranolol DSelegiline

Parkinson Disease Patient presents with rigidity, bradykinesia, postural instability, micrographia PE will show a resting pill-rolling tremor, mask-like facies, cog-wheeling of extremities, shuffling gait Most commonly caused by dopamine depletion in basal ganglia Treatment is levodopa-carbidopa

A 15-year-old G1P0 girl at 23 weeks gestation presents with sharp, left lower quadrant abdominal pain for 1 hour. She previously had an ultrasound confirming the presence of a single intrauterine pregnancy. The pain is severe and associated with nausea. Pelvic examination reveals tenderness of the left adnexa. The patient's urinalysis is unremarkable. What test should be ordered to diagnose the patient? AAbdominal X-ray BCT scan of the abdomen and pelvis CPelvic ultrasound DWhite blood cell count

Pelvic ultrasound dx: ovarian torsion

A 13-year-old boy presents to clinic with a worsening rash that started about 3 weeks ago. The patient returned from a camping trip in Pennsylvania 1 month ago. The rash started on his right forearm and has progressed to involve the right upper arm and the left arm. It is not pruritic or painful. He also reports headache and fatigue. On exam, he has several erythematous macules with central clearing on the upper extremities and trunk. Which of the following disorders is most commonly associated with the suspected diagnosis? AChronic encephalitis BComplete heart block CMonoarticular arthritis DPeripheral facial nerve palsy

Peripheral facial nerve palsy dx: early disseminated Lyme disease ------------------------------------------------------------------ vs Chronic encephalitis (A) is a finding in late Lyme disease. Symptoms include memory loss and cognitive deficits. These findings are rare in children. Complete heart block (B) is one form of carditis that may occur in early disseminated Lyme disease. However, the incidence of carditis is less frequent than the neurologic sequelae. Patients with suspected Lyme disease should have cardiac monitoring including an ECG. Arthritis (C) is the most common finding in late Lyme disease. This may occur months after the initial infection. The arthritis is usually monoarticular involving large joints. The arthritis may resolve after several weeks or may recur over time.

A male infant presents to clinic for hospital follow up on the fifth day of life. His mother reports an uneventful pregnancy with normal laboratory results. Her blood type is B+ but she is unsure of the infant's blood type. Delivery at 40 weeks gestation was by uncomplicated cesarean section for prior cesarean section. Total serum bilirubin was 8 mg/dL prior to hospital discharge on the third day of life. He is feeding a combination of breastmilk and formula every 2 to 3 hours. He has had six wet diapers in the past 24 hours and has a greenish yellow stool after each feed. Examination is notable for jaundice to the lower chest. He appears well hydrated. His weight is currently decreased 3% from birth weight. A total serum bilirubin level is 12 mg/dL. What is the most likely etiology of the neonate's jaundice? ABreast milk jaundice BIsoimmune hemolytic disease CPhysiologic jaundice DSuboptimal intake jaundice

Physiologic jaundice During the first week of life, nearly all neonates develop some degree of jaundice due to unconjugated hyperbilirubinemia. Bilirubin is a product of heme catabolism and its primary source is red blood cells. Neonates have both an increased concentration of red blood cells, as well as a shortened red blood cell lifespan. In addition, enzymes of bilirubin conjugation are less active in neonates, and enterohepatic recirculation of bilirubin is higher. All of these immature processes are normal and they result in physiologic jaundice ------------------------------------------------------------ vs Breast milk jaundice (A) typically occurs during the second week of life and results from increased enterohepatic recirculation of bilirubin. An infant at five days of age is unlikely to have breast milk jaundice. Suboptimal intake jaundice (B) typically results from suboptimal intake of breast milk during the first several days of life. Signs of dehydration may include decreased urine output, significant weight loss, persistence of meconium stools, decreased skin turgor, and excessive sleepiness. The infant does not display any of these signs.

A 25-year-old woman presents to your office with complaints of abdominal pain and constipation for the past year. She has seen a number of doctors and specialists with extensive testing revealing all normal results. She is following the dietary recommendations given to her previously, but is still only having bowel movements twice per week. Which of the following is the most appropriate next step in management? ACholestyramine BLoperamide CPolyethylene glycol DRifaximin

Polyethylene glycol ---------------------------------------------------------- vs Cholestyramine (A) is a bile acid sequestrant used to treat diarrhea. Loperamide (B) is an antidiarrheal agent used in the treatment of patients with IBS associated diarrhea. Rifaximin (D) is an antimicrobial agent that may be used in patients with severe IBS without constipation who have failed other therapies.

A 28-year-old man presents to the emergency department with lethargy and vomiting. He has a history of type I diabetes mellitus. Vital signs include heart rate 112 bpm and blood pressure 110/80 mm Hg. Laboratory analysis reveals sodium 135 mEq/L, potassium 4.0 mEq/L, chloride 100 mEq/L, bicarbonate 10 mEq/L, creatinine 1.4 mg/dL, glucose 558 mg/dL, pH 7.2. Normal saline and insulin have been ordered. Which of the following should be administered next? A5% dextrose with 0.45% saline BMagnesium CPotassium chloride DSodium bicarbonate

Potassium chloride dx: diabetic ketoacidosis Treat precipitating cause Correct volume depletion with NS, add dextrose to fluids once glucose is < 200 mg/dL Replete K+ deficit (usually falsely elevated), do not start insulin if K+ < 3.3 mEq/L IV insulin drip until anion gap closes Corrected sodium: add 1.6 mEq/L for each 100 mg/dL in serum glucose HHS = hyperglycemic hyperosmolar syndrome Diabetes-Related Ketoacidosis Patient will have diabetes History of infection, ischemia (cardiac, mesenteric), iatrogenic (e.g. steroids), insulin deficit (poor control), intoxication/illegal (cocaine abuse) (five I's) Abdominal pain, vomiting, and fatigue PE will show fruity-smelling breath, dehydration, and AMS Labs will show hyperglycemia, ketonemia, and an anion gap metabolic acidosis

A 76-year-old woman with a history of coronary artery disease presents to the emergency department complaining of recent-onset dyspnea and fatigue. Physical examination reveals peripheral edema and rales on auscultation. Which of the following additional physical exam findings would support the most likely diagnosis? ABradycardia BPulsus alternans CPulsus paradoxus DReduced jugular venous pressure EWidened pulse pressure

Pulsus alternans dx: heart failure in this patient, as it is most commonly a sign of left ventricular systolic heart failure. -------------------------------------------------------------------- vs pulsus paradoxus (C), indicated by an exaggerated fall in systolic blood pressure > 10 mm Hg during inspiration, include obstructive lung disease, cardiac tamponade, constrictive pericarditis, and pulmonary embolism. Reduced jugular venous pressure (D) reflects hypovolemia. Normal jugular venous pressure ranges from 6-8 cm H2O. Pulse pressure is defined as the difference between systolic and diastolic pressures and under normal conditions is approximately 40 mm Hg. It is directly proportional to intravascular volume change (as reflected by the stroke volume) and inversely proportional to arterial compliance. Because stroke volume is decreased in heart failure, pulse pressure will be more narrow. Widened pulse pressure (E) is found in cases of valvular disease such as aortic regurgitation, stiffening of the aorta with age, and in well-conditioned athletes.

A 6-year-old boy presents with a fever of 38.5°C, sore throat, and tender anterior cervical adenopathy. He has a cough but no runny nose. No exudate is appreciated on exam. His younger sister was treated for streptococcal pharyngitis last week and his mother would like him to be treated for streptococcal infection. According to the modified Centor criteria, which of the following is the most appropriate action? AEmpiric antibiotic treatment for streptococcal pharyngitis BRapid antigen detection testing CStreptococcal antibody titers DSymptomatic treatment without diagnostic testing ETonsillectomy after the infection resolves

Rapid antigen detection testing This patient has a modified Centor criteria score of 3 (age, fever, tender anterior cervical adenopathy) and therefore rapid antigen detection testing is warranted, with treatment if positive. Penicillin (10 days of oral therapy or one injection of intramuscular benzathine penicillin) is the treatment of choice because of cost, narrow spectrum of activity, and effectiveness. Amoxicillin is equally effective and more palatable. Erythromycin and first-generation cephalosporins are options in patients with penicillin allergy. 😨😨😨😨😨😨😨😨😨😨😨😨 Patients with a score of zero or 1 are at very low risk for streptococcal pharyngitis and do not require testing (throat culture or rapid antigen detection testing) or antibiotic therapy. Testing (using rapid antigen detection test or throat culture) is optional for patients with a score of 2 where positive results warrant antibiotic therapy. Patients with a score of 3 or higher are at high risk of streptococcal pharyngitis and should have rapid strep testing culture or both. 😨😨😨 Pharyngitis Most commonly caused by viral > bacteria (GAS, S. pyogenes) Centor criteria: estimates probability pharyngitis is streptococcal based on PE and Sx: anterior cervical lymphadenopathy, tonsillar exudate or swelling, fever, absence of cough Modified Centor criteria: +1 for age < 15 years, -1 for age > 45 years Treatment Viral: supportive Bacterial:Penicillin or amoxicillin Patients with penicillin allergy: cephalexin, cefadroxil (avoid in individuals with immediate-type hypersensitivity to penicillin), clindamycin, azithromycin, clarithromycin (resistance of group A strep to these agents is known well and varies geographically and temporally) ----------------------------------------------------------------- vs Empiric antibiotic treatment (A) ​is no longer recommended by IDSA based on symptoms alone however there is some support for treating based on a Centor criteria of 4 in the literature.

A 16-year-old adolescent girl with a history of intellectual disability presents to your clinic with her parent. The parent states that they want their daughter to have a bilateral tubal ligation after they recently discovered her looking at pornographic materials. They state that their daughter is not capable of understanding the repercussions of sexual intercourse and that they do not want her to be burdened with a child that she would not be able to raise. Upon discussions with the patient, it is clear that she is not able to understand that sexual intercourse can lead to pregnancy. What should your next step be? AExplain additional risks of sexual intercourse, including sexually transmitted infections BRefer the patient to a psychiatrist to get informed consent CRefuse the procedure because it is unlikely that the patient will get pregnant DRefuse the procedure because it violates the ethical principle of autonomy ESchedule the patient for the requested surgery

Refuse the procedure because it violates the ethical principle of autonomy four principles of medical ethics

A 4-year-old girl presents with a concern for left knee pain. The patient was started on antibiotics 1 week ago for an uncomplicated ear infection. She now has a widespread morbilliform rash, urticaria, several swollen joints, and temperature of 102.74°F (39.3°C). Which of the following is the most likely diagnosis? AImmunoglobulin A vasculitis BPostviral arthritis CSerum sickness-like illness DSystemic juvenile idiopathic arthritis

Serum Sickness Type III hypersensitivity Onset usually 7-21 days after exposure or 12-36 hours after a reexposure Flulike Sxs, fever, rash, arthralgia Treatment: withdrawal of offending agent if possible, analgesics, antihistamines, glucocorticoids (if severe)

A 22-year-old woman presents with vaginal bleeding with clots that started one hour prior to arrival. She is 10 weeks pregnant based on a previous ultrasound confirming her intrauterine pregnancy. Bedside ultrasound today does not show an intrauterine pregnancy. Her blood type is A negative and the father of the baby's blood type is not currently known. Which of the following is the best strategy for administration of Rh immunoglobulin? AA test dose should be administered first because of the risk of an anaphylactoid reaction BRh immunoglobulin 300 µg is required CRh immunoglobulin is not indicated DShe should receive Rh immunoglobulin 50 µg within 72 hours

She should receive Rh immunoglobulin 50 µg within 72 hours Rh immunization is the process by which an Rh-negative woman is exposed to Rh-positive fetal blood. When this occurs, the pregnant woman develops antibodies against the Rh-positive antigen. Rh Isoimmunization Rh-negative mothers exposed to Rh-positive blood → anti-Rh antibodies Subsequent pregnancies: jaundice, anemia, fetal hydrops, fetal death Prevention: anti-D globulin at 28 weeks (and within 72 hrs of delivery if infant is Rh+)

A 46-year-old man presents with pain in his submandibular space for 48 hours that worsens when he eats. He denies current fever or chills, but endorses "feeling dehydrated" due a recent febrile illness. Exam shows erythema and swelling of the submandibular gland and pus is easily expressed. He is handling secretions normally and phonation is unimpaired. Which of the following is the most likely cause of his symptoms? ALudwig angina BSalivary gland tumor CSialadenitis DSialolithiasis

Sialadenitis Patient presents with mouth swelling and pain that is worse with meals Most commonly caused by Staphylococcus aureus Treatment is antibiotics, warm compresses, gland massage, sialogogs ---------------------------------------------------------------- vs Ludwig angina (A) presents with edema and erythema of the external upper neck under the chin and floor of the mouth. The exam may show an upwardly displaced tongue as cellulitis and pus gather in the floor of the mouth. Phonation is muffled and tongue protrusion is typically impaired as a result. Sialolithiasis (D), or calculus formation in the salivary glands, is most common in Wharton duct, which drains the submandibular glands. While sialolithiasis also presents with post-prandial pain and local swelling, it is more likely to present with a palpable stone, particularly in Wharton duct, and less likely to allow expression of pus on exam.

An 18-year-old woman presents to the clinic for her annual preventive health examination. She is asymptomatic. There is a new palpable left-sided adnexal mass on pelvic exam. Which of the following imaging modalities is best to initially characterize the mass? AComputed tomography BMagnetic resonance imaging CPlain film radiography DSonography

Sonography dx: ovarian germ cell tumors (e.g., teratomas) and cystadenomas. A mature cystic teratoma, or dermoid cyst, is benign and the most common ovarian neoplasm in the second and third decades of life. Ovarian cystectomy is recommended for all dermoid cysts to make a definitive diagnosis, preserve ovarian tissue, and avoid potential problems, such as ovarian torsion or malignant transformation. Any persistently symptomatic ovarian neoplasm should be removed.

A 55-year-old man presents to the clinic for his annual wellness exam. He does not currently take any daily medications. A review of his vital signs reveals a height of 5 feet 10 inches, weight of 225 lbs, pulse of 79 bpm, oxygen saturation of 96%, and blood pressure of 146/92 mm Hg. A point-of-care hemoglobin A1C is performed and is 5.7%. He engages in moderate physical activity two to three times weekly and states he consumes two alcoholic drinks nightly. On physical exam, the lungs are clear to auscultation, his abdomen is soft and nontender, and an ENT exam is normal. Upon cardiac examination, his heartbeat has a regular rate and rhythm with no murmurs, gallops, or rubs. Which lifestyle intervention would provide the most benefit for this patient when discussing nonpharmacologic treatment options? ADecrease sodium intake BIncrease physical activity CReduce alcohol consumption DStart Dietary Approaches to Stop Hypertension diet EStart ketogenic diet

Start Dietary Approaches to Stop Hypertension diet Lifestyle modifications can have a significant effect on morbidity and mortality in patients at risk for cardiovascular disease. The Dietary Approaches to Stop Hypertension diet, or DASH diet, consists of four to five servings of fruit, four to five servings of vegetables, two to three servings of low-fat dairy, and < 25% fat daily. Patients who adopt the DASH diet as a lifestyle modification can reduce their average systolic blood pressure by 8-14 mm Hg. The DASH diet can also lead to weight loss, which can reduce cardiovascular risk. ----------------------------------------------------------- vs Decreased sodium intake (A) can aid in decreasing risk of cardiovascular disease, but the most benefit is found in patients who follow the DASH diet in addition to lowering sodium intake, rather than decreasing sodium intake alone. It is recommended that patients reduce their dietary sodium intake to no more than 100 mEq per day. Increasing physical activity (B) is an important component of risk reduction when it comes to cardiovascular disease. The recommendation is for patients to engage in aerobic exercise for at least 30 minutes per day, most days of the week. However, if the patient is already moderately active, increasing physical activity further is likely to have little benefit on cardiovascular risk.

A 4-day-old female born of a normal full-term pregnancy, labor, and delivery presents to the emergency department with a 1-day history of 102.2°F fever, poor feeding, irritability, and fitful sleeping. Maternal and infant history is unremarkable, with the exception that the mother had a urinary tract infection at the end of her second trimester, which was not cultured, and low-grade fever after discharge from the hospital on day 2 after delivery. Physical examination of the neonate demonstrates pallor, mild jaundice, tachycardia, episodes of apnea, difficult arousal, and a bulging fontanel. Peripheral white blood cell count is 2,000/µL, mostly lymphocytes. Examination of cerebrospinal fluid demonstrates elevated protein, low glucose, very elevated white blood cell count (98% neutrophils), and a few red blood cells. Gram stain of the cerebrospinal fluid reveals gram-positive cocci in chains. What is the most likely diagnosis? AE. coli sepsis and meningitis BStaphylococcus aureus sepsis meningitis CStreptococcus agalactiae sepsis and meningitis DStreptococcus pyogenes sepsis and meningitis

Streptococcus agalactiae sepsis and meningitis or group B Strep (GBS) major cause of perinatal bacterial infection in the United State passes to the infant during labor and delivery.

Which of the following organisms is most associated with pneumonia and bullous myringitis? ABordetella pertussis BHaemophilus influenzae CStreptococcus bovis DStreptococcus pneumoniae

Streptococcus pneumoniae

A 23-year-old woman is treated for streptococcal pharyngitis. One week later, she presents with a severe sore throat, difficulty swallowing and fever greater than 103°F. During the interview, you notice drooling and a muffled voice. She has extreme throat pain after she yawns. Oropharyngeal examination reveals anterior tonsillar pillar erythema with contralateral uvular deviation. Which of the following is the most appropriate next step in management? ACarotid endarterectomy BNeurology referral for cranial neuropathy CSurgical referral for incision and drainage DTrial of a proton-pump inhibitor

Surgical referral for incision and drainage dx: Peritonsillar abscess

A 3-year-old boy presents with a nontender left scrotal mass that has been present since birth. Scrotal mass often increases in size when he strains or sneezes. On physical exam, a soft, nontender mass that transilluminates is noted. Doppler ultrasonography shows a cystic fluid collection in the left scrotum. Which of the following is the most effective management for this patient? AAntibiotic therapy BIncision and drainage CManual detorsion DSupportive treatment ESurgical repair

Surgical repair Diagnosis of a hydrocele can be made clinically. Doppler ultrasonography is used to exclude other causes of hydroceles but may also show a cystic fluid collection in the affected scrotum. Supportive treatment of a communicating hydrocele is indicated for neonates and children younger than 2 years of age. If communicating hydroceles do not resolve by age 2 years, or if noncommunicating hydroceles become symptomatic, a surgical repair must be performed to decrease the risk of developing incarcerated inguinal hernias.

A 33-year-old wrestler presents with a worsening skin infection. He was seen in the ED two days prior and had an incision and drainage of an abscess on his left arm. The drainage has decreased but now he has an area of erythema that has expanded around his original abscess and a second purulent lesion on his leg. Which of the following is the most appropriate antibiotic choice? AAmoxicillin-clavulanic acid BCephalexin CCiprofloxacin DTrimethoprim-sulfamethoxazole

Trimethoprim-sulfamethoxazole community-associated methicillin-resistant Staph aureus (MRSA). MRSA Oxacillin minimum inhibitory concentration ≥ 4 mcg/mL Risk factors include recent hospitalization or surgery, residence in a long-term care facility or prison, HD, HIV, IVDA, sharing needles, razors, sports equipment, or towels Confirm via culture Oral options (based on local resistance patterns): clindamycin, doxycycline, linezolid, TMP-SMX IV options: daptomycin, linezolid, vancomycin Patients may be chronic carriers, consider decolonization Use good hygiene, keep equipment clean

A 65-year-old man presents with worsening dyspnea, cough, and respirophasic chest pain. He reports no past medical history. Physical exam shows decreased tactile fremitus, percussive dullness, and decreased breath sounds on the left lower lobe of the lung. Chest radiograph shows blunting of the left costophrenic angle. Which of the following is most likely to confirm the underlying etiology of the suspected diagnosis? AChest computed tomography BLateral decubitus radiograph CThoracentesis DUltrasonography EVentilation-perfusion scan

Thoracentesis dx:Pleural effusion is an abnormal accumulation of fluid in the pleural space. The pathophysiologic mechanisms of pleural effusion include increased hydrostatic or decreased oncotic pressure, abnormal capillary permeability, and decreased lymphatic clearance. There are five types of pleural effusion: exudates, transudates, empyema, hemothorax, and parapneumonic. Pleural Effusion PE will show ↓ breath sounds + dull percussion + ↓ tactile fremitus CXR will show blunting of the costophrenic angle Can also use CT or US to diagnose Most common causesTransudate: heart failureExudate: infection > malignancy, PE Management includes treating underlying cause, therapeutic thoracentesis, tube thoracostomy Light criteria are used to differentiate between transudative and exudative effusions ------------------------------------------------------------- vs Lateral decubitus radiograph (B) can help differentiate free-flowing effusion from a loculated fluid. Less than 10 mL can be visible on lateral decubitus view

A 30-year-old man with chronic rhinitis presents to the emergency room with 3 hours of epistaxis. He denies hemoptysis or a history of coagulopathy. Inspection reveals an anterior single vessel bleed. You place gauze in the affected nostril and hold compression for 10 minutes. You remove the gauze and he begins to bleed again. Use of which of the following is the most appropriate next step in managing this patient? ADouble-balloon device BElectrocautery CRibbon gauze packing DTopical 4% cocaine solution

Topical 4% cocaine solution dx:epistaxis Initial treatment is with direct compression, nasal plugging with gauze and tilting the head forward to stop blood collection in the pharynx. If bleeding does not stop, careful inspection is necessary to determine the location (anterior or posterior) and the extent (single or multiple vessels) of the bleeding. If a single anterior bleeding site is identified, the next step is to use gauze soaked in oxymetazoline or phenylephrine. If this fails, adding a 4% cocaine or lidocaine solution is recommended

A detailed history and examination does not aid in the evaluation of new-onset hypertension in a 47-year-old man. In an attempt to search for an underlying cause, you order a basic metabolic panel, complete blood count, lipid panel and ECG. Which of the following tests should also be added to this standard diagnostic screen of secondary hypertension? AAortic ultrasound BEchocardiogram CRenal angiogram DUrinalysis

Urinalysis ------------------------------------------------------------ vs Renal angiogram (C)offers great sensitivity and specificity in determining renovascular disease as a cause of secondary hypertension. However, it is not considered part of an initial evaluation, and should be considered if the initial tests above are noncontributory. Ultrasound (A) is the diagnostic test of choice when evaluating such abnormalities.Aortic coarctation or aneurysm are rare causes of secondary hypertension Echocardiography (B) may be warranted if the clinician suspects left ventricular hypertrophy, valvulopathy, coronary artery disease or prior myocardial infarction as a cause of secondary hypertension, all of which would be suggested by abnormalities found during history and physical examination.

A 71-year-old man presents to your clinic with complaints of worsening vision in dim or dark environments. On physical exam, he has bilaterally dry conjunctivae with small white patches and diffusely dry skin. The suspected diagnosis can be confirmed by testing for which one of the following vitamin deficiencies? ANiacin BThiamine CVitamin A DVitamin C

Vitamin A

A 57-year-old man presents to the ED with headache, blurred vision, and fatigue. His wife notes he has a "blood problem" that his doctor is monitoring. Which of the following underlying conditions is most frequently complicated by hyperviscosity syndrome? AAcute myeloid leukemia BMultiple myeloma CPolycythemia vera DWaldenstrom macroglobulinemia

Waldenstrom macroglobulinemia most common cause of hyperviscosity syndrome is elevated serum protein, most often secondary to Waldenstrom macroglobulinemia Hyperviscosity Syndrome Sludging → ↓ perfusion MM, Waldenström macroglobulinemia, PV, leukemias ↑ WBC, ↑ RBCs Dysproteinemias: plasmapheresis Blast transformations: leukapheresis

Which of the following physical exam findings is most consistent with thrombocytopenia? ADelayed procedural bleeding BHemarthrosis CLarge soft tissue hematoma DWet purpura

Wet purpura

A 72-year-old man living in a skilled nursing facility presents with altered mental status. His rectal temperature is 101.1°F (38.4°C). Which of the following, if found in this patient, would fulfill the criteria for systemic inflammatory response syndrome in this patient? AHeart rate 88 bpm BLactic acid 3.8 mg/dL CRespiratory rate 20/min DWhite blood cell count 13,000 cells/mm3 with 76% neutrophils, 10% lymphocytes and 12% bands

White blood cell count 13,000 cells/mm3 with 76% neutrophils, 10% lymphocytes and 12% bands *SIRS criteria include 1 point for each of the following (score range, 0-4): Fever > 38°C (> 100.4°F) or < 36°C (< 96.8°F) Tachypnea with > 20 breaths/min Tachycardia with heart rate > 90 bpm Leukocytosis with WBC count > 12,000/uL, leukopenia (< 4,000/uL) or > 10% bands Criteria (1 point for each, score ≥ 2 meets SIRS definition) T > 38°C or < 36°C RR > 20/min or PaCO2 < 32 mm Hg HR > 90 bpm WBC > 12,000/μL or < 4,000/μL or > 10% bands

A 64-year-old man presents to the emergency department with acute onset of left arm and left leg weakness approximately 1 hour ago. He has slurred speech and is difficult to understand while obtaining the history. Vital signs are HR 80 bpm, BP 192/100 mm Hg, RR 12/min, and T 37.0°C. Examination shows facial asymmetry with left upper and lower extremity weakness. What is the most important bedside diagnostic test to perform at this time? AChest radiograph BEchocardiogram CElectrocardiogram DFingerstick glucose

acute onset of focal weakness of which the differential diagnosis includes cerebrovascular accident (CVA), transient ischemic attack (TIA), hypoglycemia, seizure with Todd paralysis, complicated migraine, tumor, and functional neurologic symptom disorder. hypoglycemia, a common stroke mimic.

A 57-year-old woman is brought by ambulance to the emergency room. Three hours ago she began having a headache with nausea and noticed a loss of sensation on most of the right side of her body. A non-contrast computed tomography scan of her head shows a 2 cm area of enhancement in the brain parenchyma. Which of the following is the most likely location of the patient's lesion? ACerebellum BFrontal lobe CTemporal lobe DThalamus

thalamic hemorrhage classically presents with contralateral hemisensory loss Ischemic Stroke Anterior cerebral artery: frontal lobe dysfunction, apraxia, contralateral paralysis (lower > upper) Middle cerebral artery: contralateral paralysis (upper > lower), aphasia Posterior cerebral artery and VBI: LOC, nausea or vomiting, CN dysfunction, ataxia, visual agnosia Rule out hypoglycemia CT reveals loss of gray-white interface, acute hypodensity Thrombolytics within 4.5 hrs; possible thrombectomy within 24 hours ---------------------------------------------------------------- vs Cerebellar (A) hemorrhages most often present with symptoms of increased intracranial pressure due to compression of the fourth ventricle (lethargy, nausea, vomiting). Compression of the brainstem can also occur leading to comatose before seeing hemiparesis. Other lesions of the cerebellum can cause various forms of ataxia. Frontal lobe (B) lesions can cause a contralateral palsy due to the primary and pre-motor cortex in the frontal lobe but the primary sensory cortex is in the parietal lobe. Broca's area is also in the frontal lobe and damage would lead to an expressive aphasia (good understanding but difficulty producing comprehendible speech). A lesion in the dominant temporal lobe (C) could lead to a Wernicke's aphasia (receptive aphasia) where the patient speaks clearly but has trouble understanding.

A 23-year-old woman presents with symptoms of recurrent nosebleeds, bleeding of the gums, and menorrhagia. Significant laboratory results include a normal platelet count, abnormal platelet function analysis, normal prothrombin time, and abnormal activated partial thromboplastin time. Which of the following is the most likely diagnosis? ADisseminated intravascular coagulation BHemophilia A CThrombotic thrombocytopenic purpura Dvon Willebrand's disease

von Willebrand's disease Laboratory tests confirm the diagnosis and show normal PT, prolonged aPTT in 50% of patients with vWD (due to decreased factor VIII activity), normal platelet count with abnormal platelet function, and low vWF. Desmopressin acetate (DDAVP) Laboratory • Normal platelet count (mild thrombocytopenia in type 2B) • Normal PT/INR • Normal aPTT (prolonged if very low factor VIll) Clinical • Easy bruising • Skin bleeding • Prolonged bleeding from mucosal surfaces (oropharyngeal, GI, uterine) von Willebrand Disease Most common inherited bleeding disorder due to reduced, dysfunctional, or absent von Willebrand factor Sx: increased mucocutaneous bleeding, heavy menses, excessive postpartum bleeding Labs: aPTT may be prolonged (if factor VIII low), VWF antigen, platelet-dependent VWF activity (ristocetin cofactor assay), factor VIII activity Tx options: desmopressin (DDAVP), von Willebrand factor concentrate Most cases autosomal dominant, consider genetic counseling and testing of 1st degree relatives


Related study sets

M03-Ch.14&Ch.15- Troubleshooting Windows startup

View Set

Level I Antiterrorism Training - Pre Test

View Set

Chapter 22: Genomics I: Analysis of DNA

View Set

Jewish Feasts/Festivals Unit 2: Bible 7

View Set

Skills and Essentials Ch 28 Infection Control

View Set

2 blood types, blood transfusion and tissue and organ transplantation

View Set